Last visit was: 23 Apr 2024, 20:39 It is currently 23 Apr 2024, 20:39

Close
GMAT Club Daily Prep
Thank you for using the timer - this advanced tool can estimate your performance and suggest more practice questions. We have subscribed you to Daily Prep Questions via email.

Customized
for You

we will pick new questions that match your level based on your Timer History

Track
Your Progress

every week, we’ll send you an estimated GMAT score based on your performance

Practice
Pays

we will pick new questions that match your level based on your Timer History
Not interested in getting valuable practice questions and articles delivered to your email? No problem, unsubscribe here.
Close
Request Expert Reply
Confirm Cancel
SORT BY:
Date
Tags:
Difficulty: 555-605 Levelx   Statistics and Sets Problemsx                              
Show Tags
Hide Tags
User avatar
Manager
Manager
Joined: 02 Dec 2012
Posts: 172
Own Kudos [?]: 23844 [496]
Given Kudos: 23
Send PM
Most Helpful Reply
Math Expert
Joined: 02 Sep 2009
Posts: 92883
Own Kudos [?]: 618618 [252]
Given Kudos: 81563
Send PM
Manager
Manager
Joined: 20 Dec 2013
Posts: 183
Own Kudos [?]: 290 [45]
Given Kudos: 35
Location: India
Send PM
GMAT Club Legend
GMAT Club Legend
Joined: 12 Sep 2015
Posts: 6821
Own Kudos [?]: 29894 [28]
Given Kudos: 799
Location: Canada
Send PM
Re: If Q is an odd number and the median of Q consecutive integers is 120 [#permalink]
18
Kudos
10
Bookmarks
Expert Reply
Top Contributor
Walkabout wrote:
If Q is an odd number and the median of Q consecutive integers is 120, what is the largest of these integers?

(A) (Q - 1)/2 + 120
(B) Q/2 + 119
(C) Q/2 + 120
(D) (Q + 119)/2
(E) (Q + 120)/2


A very fast solution is to see what happens when Q = 1.
This means that there's only ONE integer in the set.
So, if the median of the set is 120, then the set is {120}, which means the greatest value in the set is 120

So the correct answer choice should yield 120 when Q = 1.

a) (1-1)/2 + 120 = 120 PERFECT!
b) 1/2 + 119 = some non-integer
c) 1/2 + 120 = some non-integer
d) (1+119)/2 = 60
e) (1+120)/2 = some non-integer

Since only answer choice A yield the correct output, it is the correct answer.

Cheers,
Brent
General Discussion
User avatar
Senior Manager
Senior Manager
Joined: 13 Aug 2012
Posts: 336
Own Kudos [?]: 1821 [8]
Given Kudos: 11
Concentration: Marketing, Finance
GPA: 3.23
Send PM
Re: If Q is an odd number and the median of Q consecutive integers is 120 [#permalink]
5
Kudos
3
Bookmarks
Walkabout wrote:
If Q is an odd number and the median of Q consecutive integers is 120, what is the largest of these integers?

(A) (Q - 1)/2 + 120
(B) Q/2 + 119
(C) Q/2 + 120
(D) (Q + 119)/2
(E) (Q + 120)/2


Let Q = 3
Set = {119,120,121} which makes 121 the largest integer

(A) (3-1)/2 + 120 = 121
(B) and (C) yields a decimal. ELIMATE!
(D) 122/2 = 61 ELIMINATE!
(E) yields a decimal. ELIMINATE!

Answer: A
avatar
Director
Director
Joined: 29 Nov 2012
Posts: 580
Own Kudos [?]: 6040 [4]
Given Kudos: 543
Send PM
Re: If Q is an odd number and the median of Q consecutive integers is 120 [#permalink]
2
Kudos
1
Bookmarks
Really nice method Bunuel!
avatar
Intern
Intern
Joined: 11 Jan 2013
Posts: 12
Own Kudos [?]: 108 [1]
Given Kudos: 12
Location: United States
Send PM
Re: If Q is an odd number and the median of Q consecutive integers is 120 [#permalink]
Bunuel wrote:
Walkabout wrote:
If Q is an odd number and the median of Q consecutive integers is 120, what is the largest of these integers?

(A) (Q - 1)/2 + 120
(B) Q/2 + 119
(C) Q/2 + 120
(D) (Q + 119)/2
(E) (Q + 120)/2


Consider the easiest case, say Q=3, then;
Set = {119, 120, 121};
The largest integer = 121.

Now, plug Q=3 into the answers to see which yields 121. Only answer choice A works. Notice that we don't really need to plug for B, C, or E, since these options do not yield an integer value for any odd value of Q.

Answer: A.


Sorry, if this is a really stupid question. I am probably missing something, but I do not understand the logic of this approach:

If 120 is the median (i.e. in my mind the middle number of the set) how can 121 then be the largest number? I am asking, since this does not make sense to me, yet and thus I would have never come up with such a good shortcut.

Many thanks
Math Expert
Joined: 02 Sep 2009
Posts: 92883
Own Kudos [?]: 618618 [4]
Given Kudos: 81563
Send PM
Re: If Q is an odd number and the median of Q consecutive integers is 120 [#permalink]
3
Kudos
1
Bookmarks
Expert Reply
Revenge2013 wrote:
Bunuel wrote:
Walkabout wrote:
If Q is an odd number and the median of Q consecutive integers is 120, what is the largest of these integers?

(A) (Q - 1)/2 + 120
(B) Q/2 + 119
(C) Q/2 + 120
(D) (Q + 119)/2
(E) (Q + 120)/2


Consider the easiest case, say Q=3, then;
Set = {119, 120, 121};
The largest integer = 121.

Now, plug Q=3 into the answers to see which yields 121. Only answer choice A works. Notice that we don't really need to plug for B, C, or E, since these options do not yield an integer value for any odd value of Q.

Answer: A.


Sorry, if this is a really stupid question. I am probably missing something, but I do not understand the logic of this approach:

If 120 is the median (i.e. in my mind the middle number of the set) how can 121 then be the largest number? I am asking, since this does not make sense to me, yet and thus I would have never come up with such a good shortcut.

Many thanks


We are told that there are Q consecutive integers in a set and Q is odd. We are also told that the median of the set is 120.

Now, say Q=3=odd. So, we have that the median of 3 consecutive integers is 120.
Question: what is the largest of these 3 integers? The set in this case must be {119, 120, 121} (3 consecutive integers with median of 120), so the largest of these 3 integers is 121.

Hope it's clear.
Current Student
Joined: 21 Sep 2013
Posts: 108
Own Kudos [?]: 32 [5]
Given Kudos: 13
Location: India
Concentration: Strategy, Sustainability
GPA: 4
WE:Consulting (Consulting)
Send PM
Re: If Q is an odd number and the median of Q consecutive integers is 120 [#permalink]
5
Kudos
I came across an alternate method.
Since Q is odd, therefore Q/2 is a fraction this options B and C are eliminated.( questions talks about integers only).E option is nullified since Q+120 is odd and the result will be a fraction on division with respect to 2. Left A and D just put Q =1,3 or any odd number D gives a value less than 120 therefore it cannot be the largest.

Posted from my mobile device
avatar
Intern
Intern
Joined: 08 Jul 2013
Posts: 2
Own Kudos [?]: 4 [4]
Given Kudos: 6
Send PM
Re: If Q is an odd number and the median of Q consecutive integers is 120 [#permalink]
4
Kudos
Walkabout wrote:
If Q is an odd number and the median of Q consecutive integers is 120, what is the largest of these integers?

(A) (Q - 1)/2 + 120
(B) Q/2 + 119
(C) Q/2 + 120
(D) (Q + 119)/2
(E) (Q + 120)/2


Average of Q consecutive ints in a list = average of first and the last ints in the list
also for consecutive int mean = median
F = First Number
l = Last Number

avg = \frac{(F + L)}{2}

now L = F + (Q-1)

\frac{(F + F +(Q-1))}{2} = 120

F = 120 - \frac{(Q-1)}{2}

L = 120 - \frac{(Q-1)}{2} + (Q-1) = 120 + \frac{(Q-1)}{2}
Intern
Intern
Joined: 21 Jun 2011
Posts: 48
Own Kudos [?]: 109 [1]
Given Kudos: 15
Location: United States
Concentration: Accounting, Finance
WE:Accounting (Accounting)
Send PM
Re: If Q is an odd number and the median of Q consecutive integers is 120 [#permalink]
1
Kudos
Bunuel wrote:
Walkabout wrote:
If Q is an odd number and the median of Q consecutive integers is 120, what is the largest of these integers?

(A) (Q - 1)/2 + 120
(B) Q/2 + 119
(C) Q/2 + 120
(D) (Q + 119)/2
(E) (Q + 120)/2


Consider the easiest case, say Q=3, then;
Set = {119, 120, 121};
The largest integer = 121.

Now, plug Q=3 into the answers to see which yields 121. Only answer choice A works. Notice that we don't really need to plug for B, C, or E, since these options do not yield an integer value for any odd value of Q.

Answer: A.


Little eager....might be stupid too but can you assume q=1. This brings me to an important question can you have a median in a set that has only one element. I know this might sound stupid but is this possible. Further if that being so my largest number through option 1 becomes 120. But that is wrong. So am I missing something or the question has been set up assuming odd number starts with 3.

Bunuel request you to please clarify this.
GMAT Club Legend
GMAT Club Legend
Joined: 03 Oct 2013
Affiliations: CrackVerbal
Posts: 4946
Own Kudos [?]: 7624 [0]
Given Kudos: 215
Location: India
Send PM
Re: If Q is an odd number and the median of Q consecutive integers is 120 [#permalink]
Walkabout wrote:
If Q is an odd number and the median of Q consecutive integers is 120, what is the largest of these integers?

(A) (Q - 1)/2 + 120
(B) Q/2 + 119
(C) Q/2 + 120
(D) (Q + 119)/2
(E) (Q + 120)/2


Let us say that the numbers are {119, 120, 121}
Q = 3

(A) (3 - 1)/2 + 120 = 121
(B) 119 + 1.5
(C) 1.5 + 120
(D) (122)/2
(E) (123)/2

Hence option A is the answer
Math Expert
Joined: 02 Sep 2009
Posts: 92883
Own Kudos [?]: 618618 [4]
Given Kudos: 81563
Send PM
Re: If Q is an odd number and the median of Q consecutive integers is 120 [#permalink]
4
Kudos
Expert Reply
davidfrank wrote:
Bunuel wrote:
Walkabout wrote:
If Q is an odd number and the median of Q consecutive integers is 120, what is the largest of these integers?

(A) (Q - 1)/2 + 120
(B) Q/2 + 119
(C) Q/2 + 120
(D) (Q + 119)/2
(E) (Q + 120)/2


Consider the easiest case, say Q=3, then;
Set = {119, 120, 121};
The largest integer = 121.

Now, plug Q=3 into the answers to see which yields 121. Only answer choice A works. Notice that we don't really need to plug for B, C, or E, since these options do not yield an integer value for any odd value of Q.

Answer: A.


Little eager....might be stupid too but can you assume q=1. This brings me to an important question can you have a median in a set that has only one element. I know this might sound stupid but is this possible. Further if that being so my largest number through option 1 becomes 120. But that is wrong. So am I missing something or the question has been set up assuming odd number starts with 3.

Bunuel request you to please clarify this.


The median of a single element set is that number itself. For example, the median of {-11} is -11.

Next, you can consider Q to be 1, in this case the set is {120} and the largest integer is 120. Substituting Q=1 into the options gives A as the answer.

Hope it's clear.
User avatar
Retired Moderator
Joined: 29 Apr 2015
Posts: 717
Own Kudos [?]: 4224 [2]
Given Kudos: 302
Location: Switzerland
Concentration: Economics, Finance
Schools: LBS MIF '19
WE:Asset Management (Investment Banking)
Send PM
Re: If Q is an odd number and the median of Q consecutive integers is 120 [#permalink]
1
Kudos
1
Bookmarks
Bunuel wrote:
Walkabout wrote:
If Q is an odd number and the median of Q consecutive integers is 120, what is the largest of these integers?

(A) (Q - 1)/2 + 120
(B) Q/2 + 119
(C) Q/2 + 120
(D) (Q + 119)/2
(E) (Q + 120)/2


Consider the easiest case, say Q=3, then;
Set = {119, 120, 121};
The largest integer = 121.

Now, plug Q=3 into the answers to see which yields 121. Only answer choice A works. Notice that we don't really need to plug for B, C, or E, since these options do not yield an integer value for any odd value of Q.

Answer: A.


Dear Bunuel
Is this question properly formulated? "If Q is an odd number and the median of Q consecutive integers is 120, what is the largest of these integers?"
Shouldn't it say "If Q is a set of consecutive numbers and Q is is odd, then wat is the largest of..."?
I find the wording a little confusing.
Math Expert
Joined: 02 Sep 2009
Posts: 92883
Own Kudos [?]: 618618 [0]
Given Kudos: 81563
Send PM
Re: If Q is an odd number and the median of Q consecutive integers is 120 [#permalink]
Expert Reply
reto wrote:
Bunuel wrote:
Walkabout wrote:
If Q is an odd number and the median of Q consecutive integers is 120, what is the largest of these integers?

(A) (Q - 1)/2 + 120
(B) Q/2 + 119
(C) Q/2 + 120
(D) (Q + 119)/2
(E) (Q + 120)/2


Consider the easiest case, say Q=3, then;
Set = {119, 120, 121};
The largest integer = 121.

Now, plug Q=3 into the answers to see which yields 121. Only answer choice A works. Notice that we don't really need to plug for B, C, or E, since these options do not yield an integer value for any odd value of Q.

Answer: A.


Dear Bunuel
Is this question properly formulated? "If Q is an odd number and the median of Q consecutive integers is 120, what is the largest of these integers?"
Shouldn't it say "If Q is a set of consecutive numbers and Q is is odd, then wat is the largest of..."?
I find the wording a little confusing.


No.

The set given is a set of "Q consecutive integers", where Q is an odd number. For example, if Q=5=odd, then the set can be {118, 119, 120, 121, 122} (set of 5 consecutive integers).
GMAT Club Legend
GMAT Club Legend
Joined: 19 Dec 2014
Status:GMAT Assassin/Co-Founder
Affiliations: EMPOWERgmat
Posts: 21846
Own Kudos [?]: 11664 [4]
Given Kudos: 450
Location: United States (CA)
GMAT 1: 800 Q51 V49
GRE 1: Q170 V170
Send PM
Re: If Q is an odd number and the median of Q consecutive integers is 120 [#permalink]
4
Kudos
Expert Reply
Hi reto,

You'll likely find it best to break Quant prompts down into 'pieces', so that you can deal with each piece one-at-a-time and simplify any complex-looking situations.

Here, the first thing we're told is "Q is an ODD number..." Now what does THAT mean? Here are some examples...

Q COULD be -5, -3, -1, 1, 3, 5, 7, etc.

Next, we're told "the median of Q consecutive integers is 120." 'Q' clearly refers to the 'Q' in the earlier part of the sentence; we're also dealing with the statistical term "median" (which means 'middle number of a group, when the numbers are ordered from least to greatest').

So let's list out some possibilities (it's worth noting that you can't have "negative consecutive integers", so the negative examples that I listed are not possible here):

IF...Q = 1....the median of 1 consecutive number is 120......so the 'group' is {120}
IF...Q = 3....the median of 3 consecutive numbers is 120....so the 'group' is {119, 120, 121}
IF...Q = 5....the median of 5 consecutive numbers is 120....so the 'group' is {118, 119, 120, 121, 122}
Etc.

GMAT assassins aren't born, they're made,
Rich
Target Test Prep Representative
Joined: 04 Mar 2011
Status:Head GMAT Instructor
Affiliations: Target Test Prep
Posts: 3043
Own Kudos [?]: 6271 [0]
Given Kudos: 1646
Send PM
Re: If Q is an odd number and the median of Q consecutive integers is 120 [#permalink]
Expert Reply
Walkabout wrote:
If Q is an odd number and the median of Q consecutive integers is 120, what is the largest of these integers?

(A) (Q - 1)/2 + 120
(B) Q/2 + 119
(C) Q/2 + 120
(D) (Q + 119)/2
(E) (Q + 120)/2


We are given that Q is an ODD NUMBER and that the median of Q CONSECUTIVE INTEGERS is 120. Let's choose a convenient number for Q, such as 3. We can now say:

The median of 3 consecutive integers is 120. Since 120 is the MEDIAN, or middle number of these integers, our 3 integers are the following:

119, 120, 121

The answer choices present us with formulas for the value of the largest integer in the sequence. To determine the correct formula, we therefore will plug 3 in for Q in each answer choice until we get 121, which is the largest value in our sample data set.

A) (Q-1)/2 + 120

(3-1)/2 + 120 = 1 + 120 = 121

This IS equal to 121.


B) Q/2 + 119

3/2 + 119 = 1.5 + 119 = 120.5

This IS NOT equal to 121.

C) Q/2 + 120

3/2 + 120 = 1.5 + 120 = 121.5

This IS NOT equal to 121.

D) (Q+119)/2

(3+119)/2 = 122/2 = 61

This IS NOT equal to 121.

E) (Q+120)/2

(3+120)/2 = 123/2 = 61.5

This IS NOT equal to 121.

The only answer that is equal to 121 is that given in answer choice A.

Answer: A
Director
Director
Joined: 21 Feb 2017
Posts: 521
Own Kudos [?]: 1034 [1]
Given Kudos: 1091
Location: India
GMAT 1: 700 Q47 V39
Send PM
Re: If Q is an odd number and the median of Q consecutive integers is 120 [#permalink]
1
Kudos
Hi, I know it's given that Q has to be an integer and basis that we eliminate B, C and D. But C gives us 121.5 cant we round this to 122 and this would be greater than A: 121?
GMAT Club Legend
GMAT Club Legend
Joined: 19 Dec 2014
Status:GMAT Assassin/Co-Founder
Affiliations: EMPOWERgmat
Posts: 21846
Own Kudos [?]: 11664 [0]
Given Kudos: 450
Location: United States (CA)
GMAT 1: 800 Q51 V49
GRE 1: Q170 V170
Send PM
Re: If Q is an odd number and the median of Q consecutive integers is 120 [#permalink]
Expert Reply
Kritisood wrote:
Hi, I know it's given that Q has to be an integer and basis that we eliminate B, C and D. But C gives us 121.5 cant we round this to 122 and this would be greater than A: 121?


Hi Kritisood,

We can certainly use Number Properties to eliminate three of the answers - since Q is an ODD integer and we're looking for an answer that is ALSO an integer (re: the largest integer in the group of CONSECUTIVE INTEGERS that the question describes) - Answers B, C and E can be eliminated because those answers will all be NON-integers.

The question asks us for the largest integer in the SEQUENCE of consecutive integers - NOT the largest result among the 5 answer choices. That's ultimately why Answer C is incorrect (even though we've already eliminated it for being a non-integer).

GMAT assassins aren't born, they're made,
Rich
Intern
Intern
Joined: 23 Oct 2020
Posts: 14
Own Kudos [?]: 0 [0]
Given Kudos: 85
Send PM
Re: If Q is an odd number and the median of Q consecutive integers is 120 [#permalink]
I have a problem with plugging in numbers , i always think that it won't work if i try different values. How do i make sure that it would be the same result everytime ?
GMAT Club Bot
Re: If Q is an odd number and the median of Q consecutive integers is 120 [#permalink]
 1   2   
Moderators:
Math Expert
92883 posts
Senior Moderator - Masters Forum
3137 posts

Powered by phpBB © phpBB Group | Emoji artwork provided by EmojiOne